Why is the solution to the equation $ln(x)+ln(3x+1) = 0$ not $x=frac{-1}{4}$?












-1












$begingroup$


Wouldn't you just move the $ln(x)$ to the other side, then raise both sides to the power of $e$? And then you have the same bases equal to each other, so you get $3x + 1 = x$? From where you'd get $x =frac{ -1}{4}$? Is there something wrong with this process?



This question was on a calculus exam so it does seem overly simple...any help is appreciated.










share|cite|improve this question











$endgroup$








  • 1




    $begingroup$
    FYI, the solution to $3x + 1 = x$ is $x = -1/2$.
    $endgroup$
    – eyeballfrog
    Dec 7 '18 at 23:55


















-1












$begingroup$


Wouldn't you just move the $ln(x)$ to the other side, then raise both sides to the power of $e$? And then you have the same bases equal to each other, so you get $3x + 1 = x$? From where you'd get $x =frac{ -1}{4}$? Is there something wrong with this process?



This question was on a calculus exam so it does seem overly simple...any help is appreciated.










share|cite|improve this question











$endgroup$








  • 1




    $begingroup$
    FYI, the solution to $3x + 1 = x$ is $x = -1/2$.
    $endgroup$
    – eyeballfrog
    Dec 7 '18 at 23:55
















-1












-1








-1





$begingroup$


Wouldn't you just move the $ln(x)$ to the other side, then raise both sides to the power of $e$? And then you have the same bases equal to each other, so you get $3x + 1 = x$? From where you'd get $x =frac{ -1}{4}$? Is there something wrong with this process?



This question was on a calculus exam so it does seem overly simple...any help is appreciated.










share|cite|improve this question











$endgroup$




Wouldn't you just move the $ln(x)$ to the other side, then raise both sides to the power of $e$? And then you have the same bases equal to each other, so you get $3x + 1 = x$? From where you'd get $x =frac{ -1}{4}$? Is there something wrong with this process?



This question was on a calculus exam so it does seem overly simple...any help is appreciated.







calculus algebra-precalculus






share|cite|improve this question















share|cite|improve this question













share|cite|improve this question




share|cite|improve this question








edited Dec 8 '18 at 7:54









gimusi

92.8k84494




92.8k84494










asked Dec 7 '18 at 23:48









James RonaldJames Ronald

1257




1257








  • 1




    $begingroup$
    FYI, the solution to $3x + 1 = x$ is $x = -1/2$.
    $endgroup$
    – eyeballfrog
    Dec 7 '18 at 23:55
















  • 1




    $begingroup$
    FYI, the solution to $3x + 1 = x$ is $x = -1/2$.
    $endgroup$
    – eyeballfrog
    Dec 7 '18 at 23:55










1




1




$begingroup$
FYI, the solution to $3x + 1 = x$ is $x = -1/2$.
$endgroup$
– eyeballfrog
Dec 7 '18 at 23:55






$begingroup$
FYI, the solution to $3x + 1 = x$ is $x = -1/2$.
$endgroup$
– eyeballfrog
Dec 7 '18 at 23:55












4 Answers
4






active

oldest

votes


















5












$begingroup$

We don't need to use esponential, indeed by $log A+ log B = log (AB)$ with the conditions $x>0 implies 3x+1>0$, we have that



$$ln x + ln(3x+1)=ln (3x^2+x)=0 iff 3x^2+x=1 iff 3x^2+x-1=0$$



then



$$x=frac{-1pm sqrt{1+12}}{6} implies x=frac{-1+ sqrt{13}}{6}$$






share|cite|improve this answer











$endgroup$





















    1












    $begingroup$

    Yes, there is and no, that's not the first thing you do. You should first impose the condition that both $x$ and $3x+1$ are elements of the domain of $ln$, and then do any algebra you desire.






    share|cite|improve this answer









    $endgroup$





















      1












      $begingroup$

      I think you made a mistake of signs when you moved the $ln x$ to the other side. It should be $ln(3x+1)=-ln(x)$.



      Now you exponentiate both sides, and get $3x+1=frac1x$.



      After you're done solving that equation, remember to check that those solutions still work with the original equation; logarithms do not like negative inputs. Or you can do what the other answer says and keep track of which values of $x$ are valid as you go, and check your solutions against that at the end.






      share|cite|improve this answer









      $endgroup$





















        1












        $begingroup$

        If you do that you get $ln x = -ln (3x+1)$



        and when you use both sides as powers of $e$ to get



        $e^{ln x} = e^{-ln (3x+1)} = frac 1{e^{3x+1}}$ so that your first mistake. Because one side negative you will get a reciprical.



        $x =frac 1{3x+1}$ so



        $x(3x+1) = 1$



        $3x^2 + x -1=0$



        So $x = frac {-1 pm sqrt{13}}6$ but $x > 0$ as logarithms of non-positives are not defined.



        So $x = frac {-1 + sqrt{13}}6$



        Alternatively you could have combined to get



        $ln x + ln(x+3) = 0$



        $ln x(x+3) = 0$ so



        $x(x+3) = 1$ and etc.






        share|cite|improve this answer











        $endgroup$













        • $begingroup$
          The solutions are wrong
          $endgroup$
          – AryanSonwatikar
          Dec 8 '18 at 0:53










        • $begingroup$
          Knew something was weird.....
          $endgroup$
          – fleablood
          Dec 8 '18 at 1:18










        • $begingroup$
          @fleablood Now its fine.
          $endgroup$
          – gimusi
          Dec 8 '18 at 7:10











        Your Answer





        StackExchange.ifUsing("editor", function () {
        return StackExchange.using("mathjaxEditing", function () {
        StackExchange.MarkdownEditor.creationCallbacks.add(function (editor, postfix) {
        StackExchange.mathjaxEditing.prepareWmdForMathJax(editor, postfix, [["$", "$"], ["\\(","\\)"]]);
        });
        });
        }, "mathjax-editing");

        StackExchange.ready(function() {
        var channelOptions = {
        tags: "".split(" "),
        id: "69"
        };
        initTagRenderer("".split(" "), "".split(" "), channelOptions);

        StackExchange.using("externalEditor", function() {
        // Have to fire editor after snippets, if snippets enabled
        if (StackExchange.settings.snippets.snippetsEnabled) {
        StackExchange.using("snippets", function() {
        createEditor();
        });
        }
        else {
        createEditor();
        }
        });

        function createEditor() {
        StackExchange.prepareEditor({
        heartbeatType: 'answer',
        autoActivateHeartbeat: false,
        convertImagesToLinks: true,
        noModals: true,
        showLowRepImageUploadWarning: true,
        reputationToPostImages: 10,
        bindNavPrevention: true,
        postfix: "",
        imageUploader: {
        brandingHtml: "Powered by u003ca class="icon-imgur-white" href="https://imgur.com/"u003eu003c/au003e",
        contentPolicyHtml: "User contributions licensed under u003ca href="https://creativecommons.org/licenses/by-sa/3.0/"u003ecc by-sa 3.0 with attribution requiredu003c/au003e u003ca href="https://stackoverflow.com/legal/content-policy"u003e(content policy)u003c/au003e",
        allowUrls: true
        },
        noCode: true, onDemand: true,
        discardSelector: ".discard-answer"
        ,immediatelyShowMarkdownHelp:true
        });


        }
        });














        draft saved

        draft discarded


















        StackExchange.ready(
        function () {
        StackExchange.openid.initPostLogin('.new-post-login', 'https%3a%2f%2fmath.stackexchange.com%2fquestions%2f3030513%2fwhy-is-the-solution-to-the-equation-lnx-ln3x1-0-not-x-frac-14%23new-answer', 'question_page');
        }
        );

        Post as a guest















        Required, but never shown

























        4 Answers
        4






        active

        oldest

        votes








        4 Answers
        4






        active

        oldest

        votes









        active

        oldest

        votes






        active

        oldest

        votes









        5












        $begingroup$

        We don't need to use esponential, indeed by $log A+ log B = log (AB)$ with the conditions $x>0 implies 3x+1>0$, we have that



        $$ln x + ln(3x+1)=ln (3x^2+x)=0 iff 3x^2+x=1 iff 3x^2+x-1=0$$



        then



        $$x=frac{-1pm sqrt{1+12}}{6} implies x=frac{-1+ sqrt{13}}{6}$$






        share|cite|improve this answer











        $endgroup$


















          5












          $begingroup$

          We don't need to use esponential, indeed by $log A+ log B = log (AB)$ with the conditions $x>0 implies 3x+1>0$, we have that



          $$ln x + ln(3x+1)=ln (3x^2+x)=0 iff 3x^2+x=1 iff 3x^2+x-1=0$$



          then



          $$x=frac{-1pm sqrt{1+12}}{6} implies x=frac{-1+ sqrt{13}}{6}$$






          share|cite|improve this answer











          $endgroup$
















            5












            5








            5





            $begingroup$

            We don't need to use esponential, indeed by $log A+ log B = log (AB)$ with the conditions $x>0 implies 3x+1>0$, we have that



            $$ln x + ln(3x+1)=ln (3x^2+x)=0 iff 3x^2+x=1 iff 3x^2+x-1=0$$



            then



            $$x=frac{-1pm sqrt{1+12}}{6} implies x=frac{-1+ sqrt{13}}{6}$$






            share|cite|improve this answer











            $endgroup$



            We don't need to use esponential, indeed by $log A+ log B = log (AB)$ with the conditions $x>0 implies 3x+1>0$, we have that



            $$ln x + ln(3x+1)=ln (3x^2+x)=0 iff 3x^2+x=1 iff 3x^2+x-1=0$$



            then



            $$x=frac{-1pm sqrt{1+12}}{6} implies x=frac{-1+ sqrt{13}}{6}$$







            share|cite|improve this answer














            share|cite|improve this answer



            share|cite|improve this answer








            edited Dec 8 '18 at 0:06

























            answered Dec 8 '18 at 0:00









            gimusigimusi

            92.8k84494




            92.8k84494























                1












                $begingroup$

                Yes, there is and no, that's not the first thing you do. You should first impose the condition that both $x$ and $3x+1$ are elements of the domain of $ln$, and then do any algebra you desire.






                share|cite|improve this answer









                $endgroup$


















                  1












                  $begingroup$

                  Yes, there is and no, that's not the first thing you do. You should first impose the condition that both $x$ and $3x+1$ are elements of the domain of $ln$, and then do any algebra you desire.






                  share|cite|improve this answer









                  $endgroup$
















                    1












                    1








                    1





                    $begingroup$

                    Yes, there is and no, that's not the first thing you do. You should first impose the condition that both $x$ and $3x+1$ are elements of the domain of $ln$, and then do any algebra you desire.






                    share|cite|improve this answer









                    $endgroup$



                    Yes, there is and no, that's not the first thing you do. You should first impose the condition that both $x$ and $3x+1$ are elements of the domain of $ln$, and then do any algebra you desire.







                    share|cite|improve this answer












                    share|cite|improve this answer



                    share|cite|improve this answer










                    answered Dec 7 '18 at 23:50









                    Saucy O'PathSaucy O'Path

                    5,9691627




                    5,9691627























                        1












                        $begingroup$

                        I think you made a mistake of signs when you moved the $ln x$ to the other side. It should be $ln(3x+1)=-ln(x)$.



                        Now you exponentiate both sides, and get $3x+1=frac1x$.



                        After you're done solving that equation, remember to check that those solutions still work with the original equation; logarithms do not like negative inputs. Or you can do what the other answer says and keep track of which values of $x$ are valid as you go, and check your solutions against that at the end.






                        share|cite|improve this answer









                        $endgroup$


















                          1












                          $begingroup$

                          I think you made a mistake of signs when you moved the $ln x$ to the other side. It should be $ln(3x+1)=-ln(x)$.



                          Now you exponentiate both sides, and get $3x+1=frac1x$.



                          After you're done solving that equation, remember to check that those solutions still work with the original equation; logarithms do not like negative inputs. Or you can do what the other answer says and keep track of which values of $x$ are valid as you go, and check your solutions against that at the end.






                          share|cite|improve this answer









                          $endgroup$
















                            1












                            1








                            1





                            $begingroup$

                            I think you made a mistake of signs when you moved the $ln x$ to the other side. It should be $ln(3x+1)=-ln(x)$.



                            Now you exponentiate both sides, and get $3x+1=frac1x$.



                            After you're done solving that equation, remember to check that those solutions still work with the original equation; logarithms do not like negative inputs. Or you can do what the other answer says and keep track of which values of $x$ are valid as you go, and check your solutions against that at the end.






                            share|cite|improve this answer









                            $endgroup$



                            I think you made a mistake of signs when you moved the $ln x$ to the other side. It should be $ln(3x+1)=-ln(x)$.



                            Now you exponentiate both sides, and get $3x+1=frac1x$.



                            After you're done solving that equation, remember to check that those solutions still work with the original equation; logarithms do not like negative inputs. Or you can do what the other answer says and keep track of which values of $x$ are valid as you go, and check your solutions against that at the end.







                            share|cite|improve this answer












                            share|cite|improve this answer



                            share|cite|improve this answer










                            answered Dec 7 '18 at 23:54









                            ArthurArthur

                            115k7116198




                            115k7116198























                                1












                                $begingroup$

                                If you do that you get $ln x = -ln (3x+1)$



                                and when you use both sides as powers of $e$ to get



                                $e^{ln x} = e^{-ln (3x+1)} = frac 1{e^{3x+1}}$ so that your first mistake. Because one side negative you will get a reciprical.



                                $x =frac 1{3x+1}$ so



                                $x(3x+1) = 1$



                                $3x^2 + x -1=0$



                                So $x = frac {-1 pm sqrt{13}}6$ but $x > 0$ as logarithms of non-positives are not defined.



                                So $x = frac {-1 + sqrt{13}}6$



                                Alternatively you could have combined to get



                                $ln x + ln(x+3) = 0$



                                $ln x(x+3) = 0$ so



                                $x(x+3) = 1$ and etc.






                                share|cite|improve this answer











                                $endgroup$













                                • $begingroup$
                                  The solutions are wrong
                                  $endgroup$
                                  – AryanSonwatikar
                                  Dec 8 '18 at 0:53










                                • $begingroup$
                                  Knew something was weird.....
                                  $endgroup$
                                  – fleablood
                                  Dec 8 '18 at 1:18










                                • $begingroup$
                                  @fleablood Now its fine.
                                  $endgroup$
                                  – gimusi
                                  Dec 8 '18 at 7:10
















                                1












                                $begingroup$

                                If you do that you get $ln x = -ln (3x+1)$



                                and when you use both sides as powers of $e$ to get



                                $e^{ln x} = e^{-ln (3x+1)} = frac 1{e^{3x+1}}$ so that your first mistake. Because one side negative you will get a reciprical.



                                $x =frac 1{3x+1}$ so



                                $x(3x+1) = 1$



                                $3x^2 + x -1=0$



                                So $x = frac {-1 pm sqrt{13}}6$ but $x > 0$ as logarithms of non-positives are not defined.



                                So $x = frac {-1 + sqrt{13}}6$



                                Alternatively you could have combined to get



                                $ln x + ln(x+3) = 0$



                                $ln x(x+3) = 0$ so



                                $x(x+3) = 1$ and etc.






                                share|cite|improve this answer











                                $endgroup$













                                • $begingroup$
                                  The solutions are wrong
                                  $endgroup$
                                  – AryanSonwatikar
                                  Dec 8 '18 at 0:53










                                • $begingroup$
                                  Knew something was weird.....
                                  $endgroup$
                                  – fleablood
                                  Dec 8 '18 at 1:18










                                • $begingroup$
                                  @fleablood Now its fine.
                                  $endgroup$
                                  – gimusi
                                  Dec 8 '18 at 7:10














                                1












                                1








                                1





                                $begingroup$

                                If you do that you get $ln x = -ln (3x+1)$



                                and when you use both sides as powers of $e$ to get



                                $e^{ln x} = e^{-ln (3x+1)} = frac 1{e^{3x+1}}$ so that your first mistake. Because one side negative you will get a reciprical.



                                $x =frac 1{3x+1}$ so



                                $x(3x+1) = 1$



                                $3x^2 + x -1=0$



                                So $x = frac {-1 pm sqrt{13}}6$ but $x > 0$ as logarithms of non-positives are not defined.



                                So $x = frac {-1 + sqrt{13}}6$



                                Alternatively you could have combined to get



                                $ln x + ln(x+3) = 0$



                                $ln x(x+3) = 0$ so



                                $x(x+3) = 1$ and etc.






                                share|cite|improve this answer











                                $endgroup$



                                If you do that you get $ln x = -ln (3x+1)$



                                and when you use both sides as powers of $e$ to get



                                $e^{ln x} = e^{-ln (3x+1)} = frac 1{e^{3x+1}}$ so that your first mistake. Because one side negative you will get a reciprical.



                                $x =frac 1{3x+1}$ so



                                $x(3x+1) = 1$



                                $3x^2 + x -1=0$



                                So $x = frac {-1 pm sqrt{13}}6$ but $x > 0$ as logarithms of non-positives are not defined.



                                So $x = frac {-1 + sqrt{13}}6$



                                Alternatively you could have combined to get



                                $ln x + ln(x+3) = 0$



                                $ln x(x+3) = 0$ so



                                $x(x+3) = 1$ and etc.







                                share|cite|improve this answer














                                share|cite|improve this answer



                                share|cite|improve this answer








                                edited Dec 8 '18 at 7:09









                                gimusi

                                92.8k84494




                                92.8k84494










                                answered Dec 8 '18 at 0:14









                                fleabloodfleablood

                                70.5k22685




                                70.5k22685












                                • $begingroup$
                                  The solutions are wrong
                                  $endgroup$
                                  – AryanSonwatikar
                                  Dec 8 '18 at 0:53










                                • $begingroup$
                                  Knew something was weird.....
                                  $endgroup$
                                  – fleablood
                                  Dec 8 '18 at 1:18










                                • $begingroup$
                                  @fleablood Now its fine.
                                  $endgroup$
                                  – gimusi
                                  Dec 8 '18 at 7:10


















                                • $begingroup$
                                  The solutions are wrong
                                  $endgroup$
                                  – AryanSonwatikar
                                  Dec 8 '18 at 0:53










                                • $begingroup$
                                  Knew something was weird.....
                                  $endgroup$
                                  – fleablood
                                  Dec 8 '18 at 1:18










                                • $begingroup$
                                  @fleablood Now its fine.
                                  $endgroup$
                                  – gimusi
                                  Dec 8 '18 at 7:10
















                                $begingroup$
                                The solutions are wrong
                                $endgroup$
                                – AryanSonwatikar
                                Dec 8 '18 at 0:53




                                $begingroup$
                                The solutions are wrong
                                $endgroup$
                                – AryanSonwatikar
                                Dec 8 '18 at 0:53












                                $begingroup$
                                Knew something was weird.....
                                $endgroup$
                                – fleablood
                                Dec 8 '18 at 1:18




                                $begingroup$
                                Knew something was weird.....
                                $endgroup$
                                – fleablood
                                Dec 8 '18 at 1:18












                                $begingroup$
                                @fleablood Now its fine.
                                $endgroup$
                                – gimusi
                                Dec 8 '18 at 7:10




                                $begingroup$
                                @fleablood Now its fine.
                                $endgroup$
                                – gimusi
                                Dec 8 '18 at 7:10


















                                draft saved

                                draft discarded




















































                                Thanks for contributing an answer to Mathematics Stack Exchange!


                                • Please be sure to answer the question. Provide details and share your research!

                                But avoid



                                • Asking for help, clarification, or responding to other answers.

                                • Making statements based on opinion; back them up with references or personal experience.


                                Use MathJax to format equations. MathJax reference.


                                To learn more, see our tips on writing great answers.




                                draft saved


                                draft discarded














                                StackExchange.ready(
                                function () {
                                StackExchange.openid.initPostLogin('.new-post-login', 'https%3a%2f%2fmath.stackexchange.com%2fquestions%2f3030513%2fwhy-is-the-solution-to-the-equation-lnx-ln3x1-0-not-x-frac-14%23new-answer', 'question_page');
                                }
                                );

                                Post as a guest















                                Required, but never shown





















































                                Required, but never shown














                                Required, but never shown












                                Required, but never shown







                                Required, but never shown

































                                Required, but never shown














                                Required, but never shown












                                Required, but never shown







                                Required, but never shown







                                Popular posts from this blog

                                Plaza Victoria

                                In PowerPoint, is there a keyboard shortcut for bulleted / numbered list?

                                How to put 3 figures in Latex with 2 figures side by side and 1 below these side by side images but in...